Is there a possible geometric method to find length of this equilateral triangle?

The name of the pictureThe name of the pictureThe name of the pictureClash Royale CLAN TAG#URR8PPP












11












$begingroup$



Problem Given that $AD parallel BC$, $|AB| = |AD|$, $angle A=120^circ$, $E$ is the midpoint of $AD$, point $F$ lies on $BD$, $triangle EFC$ is a equilateral triangle and $|AB|=4$, find the length $|EF|$.




Figure 1



Attempt At first glance, I thought it could be solved using a geometric method. I considered the law of sines/cosines, similar triangles, Pythagorean theorem, even Menelaus' theorem, however, got properties which contributed nothing to calculate $|EF|$.



What I've got after draw a line perpendicular to $BC$ through $E$




  • $triangle ABH$ and $triangle AHD$ are both equilateral triangles of length 4.

  • $triangle EFD sim triangle GEH$

  • $|EH|=2sqrt3$

Figure 2



Algebraic method Eventually, I've changed my mind to embrace algebra. I found it is easy to coordinate $E,A,B,D$ and $C$ is related to $F$ (rotation) and $B$ (same horizontal line). Make $E$ as the origin, $AD$ points to $x$-axis, $HE$ points to $y$-axis, we got



  • $E = (0,0)$

  • $A = (-2,0)$

  • $B = (-4,-2sqrt3)$

  • $D = (2,0)$

Point $(x, y)$ in line $BD$ has $y=frac1sqrt3(x-2)$. Assume $F=(x_0,y_0)$, $C=(x_1,y_1)$, we can obtain $C$ by rotating $F$ around pivot $E$ $60^circ$ counter-clockwise



$$
beginbmatrix
x_1 \ y_1
endbmatrix
=
beginbmatrix
costheta & -sintheta \
sintheta & costheta
endbmatrix
beginbmatrix
x_0 \ y_0
endbmatrix
$$

, also we know that $BC$ is parallel to $x$-axis, then
$$
beginalign*
y_1
& = sin60^circ x_0 + cos60^circ y_0 \
& = sin60^circ x_0 + cos60^circ frac1sqrt3(x_0-2) \
& = -2sqrt3
endalign*
$$

, thus $F=(-frac52, -frac3sqrt32)$, and finally $|EF|=sqrt13$



Thoughts afterwords I noticed that $F$ (through its coordinate) is actually the midpoint of $BK$. It may be a key point in geometric method, but I cannot prove it either.



Graph I made it in GeoGebra and it is shared. Please go and edit it to save your time if you have any idea.
Link: https://www.geogebra.org/graphing/yqhbzdem










share|cite|improve this question











$endgroup$
















    11












    $begingroup$



    Problem Given that $AD parallel BC$, $|AB| = |AD|$, $angle A=120^circ$, $E$ is the midpoint of $AD$, point $F$ lies on $BD$, $triangle EFC$ is a equilateral triangle and $|AB|=4$, find the length $|EF|$.




    Figure 1



    Attempt At first glance, I thought it could be solved using a geometric method. I considered the law of sines/cosines, similar triangles, Pythagorean theorem, even Menelaus' theorem, however, got properties which contributed nothing to calculate $|EF|$.



    What I've got after draw a line perpendicular to $BC$ through $E$




    • $triangle ABH$ and $triangle AHD$ are both equilateral triangles of length 4.

    • $triangle EFD sim triangle GEH$

    • $|EH|=2sqrt3$

    Figure 2



    Algebraic method Eventually, I've changed my mind to embrace algebra. I found it is easy to coordinate $E,A,B,D$ and $C$ is related to $F$ (rotation) and $B$ (same horizontal line). Make $E$ as the origin, $AD$ points to $x$-axis, $HE$ points to $y$-axis, we got



    • $E = (0,0)$

    • $A = (-2,0)$

    • $B = (-4,-2sqrt3)$

    • $D = (2,0)$

    Point $(x, y)$ in line $BD$ has $y=frac1sqrt3(x-2)$. Assume $F=(x_0,y_0)$, $C=(x_1,y_1)$, we can obtain $C$ by rotating $F$ around pivot $E$ $60^circ$ counter-clockwise



    $$
    beginbmatrix
    x_1 \ y_1
    endbmatrix
    =
    beginbmatrix
    costheta & -sintheta \
    sintheta & costheta
    endbmatrix
    beginbmatrix
    x_0 \ y_0
    endbmatrix
    $$

    , also we know that $BC$ is parallel to $x$-axis, then
    $$
    beginalign*
    y_1
    & = sin60^circ x_0 + cos60^circ y_0 \
    & = sin60^circ x_0 + cos60^circ frac1sqrt3(x_0-2) \
    & = -2sqrt3
    endalign*
    $$

    , thus $F=(-frac52, -frac3sqrt32)$, and finally $|EF|=sqrt13$



    Thoughts afterwords I noticed that $F$ (through its coordinate) is actually the midpoint of $BK$. It may be a key point in geometric method, but I cannot prove it either.



    Graph I made it in GeoGebra and it is shared. Please go and edit it to save your time if you have any idea.
    Link: https://www.geogebra.org/graphing/yqhbzdem










    share|cite|improve this question











    $endgroup$














      11












      11








      11





      $begingroup$



      Problem Given that $AD parallel BC$, $|AB| = |AD|$, $angle A=120^circ$, $E$ is the midpoint of $AD$, point $F$ lies on $BD$, $triangle EFC$ is a equilateral triangle and $|AB|=4$, find the length $|EF|$.




      Figure 1



      Attempt At first glance, I thought it could be solved using a geometric method. I considered the law of sines/cosines, similar triangles, Pythagorean theorem, even Menelaus' theorem, however, got properties which contributed nothing to calculate $|EF|$.



      What I've got after draw a line perpendicular to $BC$ through $E$




      • $triangle ABH$ and $triangle AHD$ are both equilateral triangles of length 4.

      • $triangle EFD sim triangle GEH$

      • $|EH|=2sqrt3$

      Figure 2



      Algebraic method Eventually, I've changed my mind to embrace algebra. I found it is easy to coordinate $E,A,B,D$ and $C$ is related to $F$ (rotation) and $B$ (same horizontal line). Make $E$ as the origin, $AD$ points to $x$-axis, $HE$ points to $y$-axis, we got



      • $E = (0,0)$

      • $A = (-2,0)$

      • $B = (-4,-2sqrt3)$

      • $D = (2,0)$

      Point $(x, y)$ in line $BD$ has $y=frac1sqrt3(x-2)$. Assume $F=(x_0,y_0)$, $C=(x_1,y_1)$, we can obtain $C$ by rotating $F$ around pivot $E$ $60^circ$ counter-clockwise



      $$
      beginbmatrix
      x_1 \ y_1
      endbmatrix
      =
      beginbmatrix
      costheta & -sintheta \
      sintheta & costheta
      endbmatrix
      beginbmatrix
      x_0 \ y_0
      endbmatrix
      $$

      , also we know that $BC$ is parallel to $x$-axis, then
      $$
      beginalign*
      y_1
      & = sin60^circ x_0 + cos60^circ y_0 \
      & = sin60^circ x_0 + cos60^circ frac1sqrt3(x_0-2) \
      & = -2sqrt3
      endalign*
      $$

      , thus $F=(-frac52, -frac3sqrt32)$, and finally $|EF|=sqrt13$



      Thoughts afterwords I noticed that $F$ (through its coordinate) is actually the midpoint of $BK$. It may be a key point in geometric method, but I cannot prove it either.



      Graph I made it in GeoGebra and it is shared. Please go and edit it to save your time if you have any idea.
      Link: https://www.geogebra.org/graphing/yqhbzdem










      share|cite|improve this question











      $endgroup$





      Problem Given that $AD parallel BC$, $|AB| = |AD|$, $angle A=120^circ$, $E$ is the midpoint of $AD$, point $F$ lies on $BD$, $triangle EFC$ is a equilateral triangle and $|AB|=4$, find the length $|EF|$.




      Figure 1



      Attempt At first glance, I thought it could be solved using a geometric method. I considered the law of sines/cosines, similar triangles, Pythagorean theorem, even Menelaus' theorem, however, got properties which contributed nothing to calculate $|EF|$.



      What I've got after draw a line perpendicular to $BC$ through $E$




      • $triangle ABH$ and $triangle AHD$ are both equilateral triangles of length 4.

      • $triangle EFD sim triangle GEH$

      • $|EH|=2sqrt3$

      Figure 2



      Algebraic method Eventually, I've changed my mind to embrace algebra. I found it is easy to coordinate $E,A,B,D$ and $C$ is related to $F$ (rotation) and $B$ (same horizontal line). Make $E$ as the origin, $AD$ points to $x$-axis, $HE$ points to $y$-axis, we got



      • $E = (0,0)$

      • $A = (-2,0)$

      • $B = (-4,-2sqrt3)$

      • $D = (2,0)$

      Point $(x, y)$ in line $BD$ has $y=frac1sqrt3(x-2)$. Assume $F=(x_0,y_0)$, $C=(x_1,y_1)$, we can obtain $C$ by rotating $F$ around pivot $E$ $60^circ$ counter-clockwise



      $$
      beginbmatrix
      x_1 \ y_1
      endbmatrix
      =
      beginbmatrix
      costheta & -sintheta \
      sintheta & costheta
      endbmatrix
      beginbmatrix
      x_0 \ y_0
      endbmatrix
      $$

      , also we know that $BC$ is parallel to $x$-axis, then
      $$
      beginalign*
      y_1
      & = sin60^circ x_0 + cos60^circ y_0 \
      & = sin60^circ x_0 + cos60^circ frac1sqrt3(x_0-2) \
      & = -2sqrt3
      endalign*
      $$

      , thus $F=(-frac52, -frac3sqrt32)$, and finally $|EF|=sqrt13$



      Thoughts afterwords I noticed that $F$ (through its coordinate) is actually the midpoint of $BK$. It may be a key point in geometric method, but I cannot prove it either.



      Graph I made it in GeoGebra and it is shared. Please go and edit it to save your time if you have any idea.
      Link: https://www.geogebra.org/graphing/yqhbzdem







      geometry vectors euclidean-geometry analytic-geometry geometric-transformation






      share|cite|improve this question















      share|cite|improve this question













      share|cite|improve this question




      share|cite|improve this question








      edited Jan 28 at 5:33







      alex4814

















      asked Jan 26 at 15:33









      alex4814alex4814

      1225




      1225




















          4 Answers
          4






          active

          oldest

          votes


















          14












          $begingroup$

          Since $$angle EDF = 1over 2angle FCE $$ we see that $D$ is on a circle with center at $C$ and radius $CE =CF$ so $CD=CE$.



          enter image description here



          If $M$ is midpoint of $ED$ we have $$CE^2 = ME^2+CM^2 = 1+AG^2 = 13$$



          so $CE = sqrt13$.






          share|cite|improve this answer









          $endgroup$












          • $begingroup$
            Extremely beautiful! +1.
            $endgroup$
            – Michael Rozenberg
            Jan 27 at 5:27






          • 1




            $begingroup$
            I think the condition "$angle EDF = 1over 2angle FCE$" is NOT strong enough to guarantee D lies on the same circle that is centered at C with radius = CF.
            $endgroup$
            – Mick
            Jan 28 at 3:54










          • $begingroup$
            At least explain why the first assertion is true. The explanation takes care to list the obvious radius equality, but doesn't even explain the logic of the circle.
            $endgroup$
            – C Perkins
            Jan 28 at 5:06










          • $begingroup$
            Yes, just that it is not. But if you read further you see $CF=CE$! @Mick @C Perkins
            $endgroup$
            – greedoid
            Jan 28 at 9:03







          • 1




            $begingroup$
            That clarifies one thing. Another requirement we need to say is both C and D should be on the same side of the line EF, but the original diagram clearly showed they are.
            $endgroup$
            – Mick
            Jan 28 at 17:28


















          9












          $begingroup$

          Let $P$ be the perpendicular foot of $E$ to $BD$. We find that $|EP|=sin(angle EDP )cdot|ED|=1$.
          enter image description here

          We also find that $triangle EPF$ is congruent to $triangle CHE $ implying that
          $$
          |EP|=|CH|=1.
          $$
          By Pythagorean theorem, it follows
          $$
          |EC|^2 =|EH|^2 +|CH|^2 =13,
          $$
          i.e. $$
          |EF|=|EC| =sqrt13.
          $$






          share|cite|improve this answer











          $endgroup$












          • $begingroup$
            Can you post a diagram? I cannot visually follow this at all.
            $endgroup$
            – The Great Duck
            Jan 26 at 17:47







          • 1




            $begingroup$
            @TheGreatDuck I've added a figure. I hope this will help.
            $endgroup$
            – Song
            Jan 26 at 18:19






          • 1




            $begingroup$
            This makes sense (+1), but it would help if the diagram didn't show quite so many unneeded points and lines. CD is useless-but-distracting even though mentioned in the problem, and the intersection points I, J, K, L, M, N, O are completely irrelevant.
            $endgroup$
            – Henning Makholm
            Jan 26 at 19:57










          • $begingroup$
            @HenningMakholm Thank you for suggesting, sir. I've updated the figure. I hope this is better ..
            $endgroup$
            – Song
            Jan 26 at 22:38










          • $begingroup$
            Much nicer this way.
            $endgroup$
            – Henning Makholm
            Jan 26 at 23:49


















          3












          $begingroup$

          I like the following way.



          Let $vecAB=veca$, $vecAD=vecb$, $vecBF=pvecBD$ and $vecBC=kvecAD.$



          Thus, $$vecFE=-p(-veca+vecb)-veca+frac12vecb=(p-1)veca+left(frac12-pright)vecb$$ and
          $$vecFC=-p(-veca+vecb)+kvecb=pveca+(k-p)vecb.$$



          Now, we obtain the following system:
          $$|vecFE|=|vecFC|$$ and
          $$fracvecFEcdot vecFC=frac12$$
          with variables $p$ and $k$.



          We can solve this system and the rest is smooth.






          share|cite|improve this answer









          $endgroup$




















            2












            $begingroup$

            Let $alpha=angle DEC$. We can apply the sine law to triangle $FED$:
            $$
            EDoversin(90°-alpha)=EFoversin30°=FDoversin(alpha+60°),
            $$

            that is:
            $$
            EF=1overcosalphaquadtextandquad FD=2overcosalphasin(alpha+60°).
            $$

            Applying then the sine law to triangle $BFC$ one gets:
            $$
            FB=2overcosalphasin(alpha-60°)=4sqrt3-FD=4sqrt3-2overcosalphasin(alpha+60°).
            $$

            From this it follows $tanalpha=2sqrt3$ and $EF^2=1/cos^2alpha=1+tan^2alpha=13$.






            share|cite|improve this answer









            $endgroup$












              Your Answer





              StackExchange.ifUsing("editor", function ()
              return StackExchange.using("mathjaxEditing", function ()
              StackExchange.MarkdownEditor.creationCallbacks.add(function (editor, postfix)
              StackExchange.mathjaxEditing.prepareWmdForMathJax(editor, postfix, [["$", "$"], ["\\(","\\)"]]);
              );
              );
              , "mathjax-editing");

              StackExchange.ready(function()
              var channelOptions =
              tags: "".split(" "),
              id: "69"
              ;
              initTagRenderer("".split(" "), "".split(" "), channelOptions);

              StackExchange.using("externalEditor", function()
              // Have to fire editor after snippets, if snippets enabled
              if (StackExchange.settings.snippets.snippetsEnabled)
              StackExchange.using("snippets", function()
              createEditor();
              );

              else
              createEditor();

              );

              function createEditor()
              StackExchange.prepareEditor(
              heartbeatType: 'answer',
              autoActivateHeartbeat: false,
              convertImagesToLinks: true,
              noModals: true,
              showLowRepImageUploadWarning: true,
              reputationToPostImages: 10,
              bindNavPrevention: true,
              postfix: "",
              imageUploader:
              brandingHtml: "Powered by u003ca class="icon-imgur-white" href="https://imgur.com/"u003eu003c/au003e",
              contentPolicyHtml: "User contributions licensed under u003ca href="https://creativecommons.org/licenses/by-sa/3.0/"u003ecc by-sa 3.0 with attribution requiredu003c/au003e u003ca href="https://stackoverflow.com/legal/content-policy"u003e(content policy)u003c/au003e",
              allowUrls: true
              ,
              noCode: true, onDemand: true,
              discardSelector: ".discard-answer"
              ,immediatelyShowMarkdownHelp:true
              );



              );













              draft saved

              draft discarded


















              StackExchange.ready(
              function ()
              StackExchange.openid.initPostLogin('.new-post-login', 'https%3a%2f%2fmath.stackexchange.com%2fquestions%2f3088383%2fis-there-a-possible-geometric-method-to-find-length-of-this-equilateral-triangle%23new-answer', 'question_page');

              );

              Post as a guest















              Required, but never shown

























              4 Answers
              4






              active

              oldest

              votes








              4 Answers
              4






              active

              oldest

              votes









              active

              oldest

              votes






              active

              oldest

              votes









              14












              $begingroup$

              Since $$angle EDF = 1over 2angle FCE $$ we see that $D$ is on a circle with center at $C$ and radius $CE =CF$ so $CD=CE$.



              enter image description here



              If $M$ is midpoint of $ED$ we have $$CE^2 = ME^2+CM^2 = 1+AG^2 = 13$$



              so $CE = sqrt13$.






              share|cite|improve this answer









              $endgroup$












              • $begingroup$
                Extremely beautiful! +1.
                $endgroup$
                – Michael Rozenberg
                Jan 27 at 5:27






              • 1




                $begingroup$
                I think the condition "$angle EDF = 1over 2angle FCE$" is NOT strong enough to guarantee D lies on the same circle that is centered at C with radius = CF.
                $endgroup$
                – Mick
                Jan 28 at 3:54










              • $begingroup$
                At least explain why the first assertion is true. The explanation takes care to list the obvious radius equality, but doesn't even explain the logic of the circle.
                $endgroup$
                – C Perkins
                Jan 28 at 5:06










              • $begingroup$
                Yes, just that it is not. But if you read further you see $CF=CE$! @Mick @C Perkins
                $endgroup$
                – greedoid
                Jan 28 at 9:03







              • 1




                $begingroup$
                That clarifies one thing. Another requirement we need to say is both C and D should be on the same side of the line EF, but the original diagram clearly showed they are.
                $endgroup$
                – Mick
                Jan 28 at 17:28















              14












              $begingroup$

              Since $$angle EDF = 1over 2angle FCE $$ we see that $D$ is on a circle with center at $C$ and radius $CE =CF$ so $CD=CE$.



              enter image description here



              If $M$ is midpoint of $ED$ we have $$CE^2 = ME^2+CM^2 = 1+AG^2 = 13$$



              so $CE = sqrt13$.






              share|cite|improve this answer









              $endgroup$












              • $begingroup$
                Extremely beautiful! +1.
                $endgroup$
                – Michael Rozenberg
                Jan 27 at 5:27






              • 1




                $begingroup$
                I think the condition "$angle EDF = 1over 2angle FCE$" is NOT strong enough to guarantee D lies on the same circle that is centered at C with radius = CF.
                $endgroup$
                – Mick
                Jan 28 at 3:54










              • $begingroup$
                At least explain why the first assertion is true. The explanation takes care to list the obvious radius equality, but doesn't even explain the logic of the circle.
                $endgroup$
                – C Perkins
                Jan 28 at 5:06










              • $begingroup$
                Yes, just that it is not. But if you read further you see $CF=CE$! @Mick @C Perkins
                $endgroup$
                – greedoid
                Jan 28 at 9:03







              • 1




                $begingroup$
                That clarifies one thing. Another requirement we need to say is both C and D should be on the same side of the line EF, but the original diagram clearly showed they are.
                $endgroup$
                – Mick
                Jan 28 at 17:28













              14












              14








              14





              $begingroup$

              Since $$angle EDF = 1over 2angle FCE $$ we see that $D$ is on a circle with center at $C$ and radius $CE =CF$ so $CD=CE$.



              enter image description here



              If $M$ is midpoint of $ED$ we have $$CE^2 = ME^2+CM^2 = 1+AG^2 = 13$$



              so $CE = sqrt13$.






              share|cite|improve this answer









              $endgroup$



              Since $$angle EDF = 1over 2angle FCE $$ we see that $D$ is on a circle with center at $C$ and radius $CE =CF$ so $CD=CE$.



              enter image description here



              If $M$ is midpoint of $ED$ we have $$CE^2 = ME^2+CM^2 = 1+AG^2 = 13$$



              so $CE = sqrt13$.







              share|cite|improve this answer












              share|cite|improve this answer



              share|cite|improve this answer










              answered Jan 26 at 20:41









              greedoidgreedoid

              42.5k1153105




              42.5k1153105











              • $begingroup$
                Extremely beautiful! +1.
                $endgroup$
                – Michael Rozenberg
                Jan 27 at 5:27






              • 1




                $begingroup$
                I think the condition "$angle EDF = 1over 2angle FCE$" is NOT strong enough to guarantee D lies on the same circle that is centered at C with radius = CF.
                $endgroup$
                – Mick
                Jan 28 at 3:54










              • $begingroup$
                At least explain why the first assertion is true. The explanation takes care to list the obvious radius equality, but doesn't even explain the logic of the circle.
                $endgroup$
                – C Perkins
                Jan 28 at 5:06










              • $begingroup$
                Yes, just that it is not. But if you read further you see $CF=CE$! @Mick @C Perkins
                $endgroup$
                – greedoid
                Jan 28 at 9:03







              • 1




                $begingroup$
                That clarifies one thing. Another requirement we need to say is both C and D should be on the same side of the line EF, but the original diagram clearly showed they are.
                $endgroup$
                – Mick
                Jan 28 at 17:28
















              • $begingroup$
                Extremely beautiful! +1.
                $endgroup$
                – Michael Rozenberg
                Jan 27 at 5:27






              • 1




                $begingroup$
                I think the condition "$angle EDF = 1over 2angle FCE$" is NOT strong enough to guarantee D lies on the same circle that is centered at C with radius = CF.
                $endgroup$
                – Mick
                Jan 28 at 3:54










              • $begingroup$
                At least explain why the first assertion is true. The explanation takes care to list the obvious radius equality, but doesn't even explain the logic of the circle.
                $endgroup$
                – C Perkins
                Jan 28 at 5:06










              • $begingroup$
                Yes, just that it is not. But if you read further you see $CF=CE$! @Mick @C Perkins
                $endgroup$
                – greedoid
                Jan 28 at 9:03







              • 1




                $begingroup$
                That clarifies one thing. Another requirement we need to say is both C and D should be on the same side of the line EF, but the original diagram clearly showed they are.
                $endgroup$
                – Mick
                Jan 28 at 17:28















              $begingroup$
              Extremely beautiful! +1.
              $endgroup$
              – Michael Rozenberg
              Jan 27 at 5:27




              $begingroup$
              Extremely beautiful! +1.
              $endgroup$
              – Michael Rozenberg
              Jan 27 at 5:27




              1




              1




              $begingroup$
              I think the condition "$angle EDF = 1over 2angle FCE$" is NOT strong enough to guarantee D lies on the same circle that is centered at C with radius = CF.
              $endgroup$
              – Mick
              Jan 28 at 3:54




              $begingroup$
              I think the condition "$angle EDF = 1over 2angle FCE$" is NOT strong enough to guarantee D lies on the same circle that is centered at C with radius = CF.
              $endgroup$
              – Mick
              Jan 28 at 3:54












              $begingroup$
              At least explain why the first assertion is true. The explanation takes care to list the obvious radius equality, but doesn't even explain the logic of the circle.
              $endgroup$
              – C Perkins
              Jan 28 at 5:06




              $begingroup$
              At least explain why the first assertion is true. The explanation takes care to list the obvious radius equality, but doesn't even explain the logic of the circle.
              $endgroup$
              – C Perkins
              Jan 28 at 5:06












              $begingroup$
              Yes, just that it is not. But if you read further you see $CF=CE$! @Mick @C Perkins
              $endgroup$
              – greedoid
              Jan 28 at 9:03





              $begingroup$
              Yes, just that it is not. But if you read further you see $CF=CE$! @Mick @C Perkins
              $endgroup$
              – greedoid
              Jan 28 at 9:03





              1




              1




              $begingroup$
              That clarifies one thing. Another requirement we need to say is both C and D should be on the same side of the line EF, but the original diagram clearly showed they are.
              $endgroup$
              – Mick
              Jan 28 at 17:28




              $begingroup$
              That clarifies one thing. Another requirement we need to say is both C and D should be on the same side of the line EF, but the original diagram clearly showed they are.
              $endgroup$
              – Mick
              Jan 28 at 17:28











              9












              $begingroup$

              Let $P$ be the perpendicular foot of $E$ to $BD$. We find that $|EP|=sin(angle EDP )cdot|ED|=1$.
              enter image description here

              We also find that $triangle EPF$ is congruent to $triangle CHE $ implying that
              $$
              |EP|=|CH|=1.
              $$
              By Pythagorean theorem, it follows
              $$
              |EC|^2 =|EH|^2 +|CH|^2 =13,
              $$
              i.e. $$
              |EF|=|EC| =sqrt13.
              $$






              share|cite|improve this answer











              $endgroup$












              • $begingroup$
                Can you post a diagram? I cannot visually follow this at all.
                $endgroup$
                – The Great Duck
                Jan 26 at 17:47







              • 1




                $begingroup$
                @TheGreatDuck I've added a figure. I hope this will help.
                $endgroup$
                – Song
                Jan 26 at 18:19






              • 1




                $begingroup$
                This makes sense (+1), but it would help if the diagram didn't show quite so many unneeded points and lines. CD is useless-but-distracting even though mentioned in the problem, and the intersection points I, J, K, L, M, N, O are completely irrelevant.
                $endgroup$
                – Henning Makholm
                Jan 26 at 19:57










              • $begingroup$
                @HenningMakholm Thank you for suggesting, sir. I've updated the figure. I hope this is better ..
                $endgroup$
                – Song
                Jan 26 at 22:38










              • $begingroup$
                Much nicer this way.
                $endgroup$
                – Henning Makholm
                Jan 26 at 23:49















              9












              $begingroup$

              Let $P$ be the perpendicular foot of $E$ to $BD$. We find that $|EP|=sin(angle EDP )cdot|ED|=1$.
              enter image description here

              We also find that $triangle EPF$ is congruent to $triangle CHE $ implying that
              $$
              |EP|=|CH|=1.
              $$
              By Pythagorean theorem, it follows
              $$
              |EC|^2 =|EH|^2 +|CH|^2 =13,
              $$
              i.e. $$
              |EF|=|EC| =sqrt13.
              $$






              share|cite|improve this answer











              $endgroup$












              • $begingroup$
                Can you post a diagram? I cannot visually follow this at all.
                $endgroup$
                – The Great Duck
                Jan 26 at 17:47







              • 1




                $begingroup$
                @TheGreatDuck I've added a figure. I hope this will help.
                $endgroup$
                – Song
                Jan 26 at 18:19






              • 1




                $begingroup$
                This makes sense (+1), but it would help if the diagram didn't show quite so many unneeded points and lines. CD is useless-but-distracting even though mentioned in the problem, and the intersection points I, J, K, L, M, N, O are completely irrelevant.
                $endgroup$
                – Henning Makholm
                Jan 26 at 19:57










              • $begingroup$
                @HenningMakholm Thank you for suggesting, sir. I've updated the figure. I hope this is better ..
                $endgroup$
                – Song
                Jan 26 at 22:38










              • $begingroup$
                Much nicer this way.
                $endgroup$
                – Henning Makholm
                Jan 26 at 23:49













              9












              9








              9





              $begingroup$

              Let $P$ be the perpendicular foot of $E$ to $BD$. We find that $|EP|=sin(angle EDP )cdot|ED|=1$.
              enter image description here

              We also find that $triangle EPF$ is congruent to $triangle CHE $ implying that
              $$
              |EP|=|CH|=1.
              $$
              By Pythagorean theorem, it follows
              $$
              |EC|^2 =|EH|^2 +|CH|^2 =13,
              $$
              i.e. $$
              |EF|=|EC| =sqrt13.
              $$






              share|cite|improve this answer











              $endgroup$



              Let $P$ be the perpendicular foot of $E$ to $BD$. We find that $|EP|=sin(angle EDP )cdot|ED|=1$.
              enter image description here

              We also find that $triangle EPF$ is congruent to $triangle CHE $ implying that
              $$
              |EP|=|CH|=1.
              $$
              By Pythagorean theorem, it follows
              $$
              |EC|^2 =|EH|^2 +|CH|^2 =13,
              $$
              i.e. $$
              |EF|=|EC| =sqrt13.
              $$







              share|cite|improve this answer














              share|cite|improve this answer



              share|cite|improve this answer








              edited Jan 26 at 22:23

























              answered Jan 26 at 16:15









              SongSong

              13.2k632




              13.2k632











              • $begingroup$
                Can you post a diagram? I cannot visually follow this at all.
                $endgroup$
                – The Great Duck
                Jan 26 at 17:47







              • 1




                $begingroup$
                @TheGreatDuck I've added a figure. I hope this will help.
                $endgroup$
                – Song
                Jan 26 at 18:19






              • 1




                $begingroup$
                This makes sense (+1), but it would help if the diagram didn't show quite so many unneeded points and lines. CD is useless-but-distracting even though mentioned in the problem, and the intersection points I, J, K, L, M, N, O are completely irrelevant.
                $endgroup$
                – Henning Makholm
                Jan 26 at 19:57










              • $begingroup$
                @HenningMakholm Thank you for suggesting, sir. I've updated the figure. I hope this is better ..
                $endgroup$
                – Song
                Jan 26 at 22:38










              • $begingroup$
                Much nicer this way.
                $endgroup$
                – Henning Makholm
                Jan 26 at 23:49
















              • $begingroup$
                Can you post a diagram? I cannot visually follow this at all.
                $endgroup$
                – The Great Duck
                Jan 26 at 17:47







              • 1




                $begingroup$
                @TheGreatDuck I've added a figure. I hope this will help.
                $endgroup$
                – Song
                Jan 26 at 18:19






              • 1




                $begingroup$
                This makes sense (+1), but it would help if the diagram didn't show quite so many unneeded points and lines. CD is useless-but-distracting even though mentioned in the problem, and the intersection points I, J, K, L, M, N, O are completely irrelevant.
                $endgroup$
                – Henning Makholm
                Jan 26 at 19:57










              • $begingroup$
                @HenningMakholm Thank you for suggesting, sir. I've updated the figure. I hope this is better ..
                $endgroup$
                – Song
                Jan 26 at 22:38










              • $begingroup$
                Much nicer this way.
                $endgroup$
                – Henning Makholm
                Jan 26 at 23:49















              $begingroup$
              Can you post a diagram? I cannot visually follow this at all.
              $endgroup$
              – The Great Duck
              Jan 26 at 17:47





              $begingroup$
              Can you post a diagram? I cannot visually follow this at all.
              $endgroup$
              – The Great Duck
              Jan 26 at 17:47





              1




              1




              $begingroup$
              @TheGreatDuck I've added a figure. I hope this will help.
              $endgroup$
              – Song
              Jan 26 at 18:19




              $begingroup$
              @TheGreatDuck I've added a figure. I hope this will help.
              $endgroup$
              – Song
              Jan 26 at 18:19




              1




              1




              $begingroup$
              This makes sense (+1), but it would help if the diagram didn't show quite so many unneeded points and lines. CD is useless-but-distracting even though mentioned in the problem, and the intersection points I, J, K, L, M, N, O are completely irrelevant.
              $endgroup$
              – Henning Makholm
              Jan 26 at 19:57




              $begingroup$
              This makes sense (+1), but it would help if the diagram didn't show quite so many unneeded points and lines. CD is useless-but-distracting even though mentioned in the problem, and the intersection points I, J, K, L, M, N, O are completely irrelevant.
              $endgroup$
              – Henning Makholm
              Jan 26 at 19:57












              $begingroup$
              @HenningMakholm Thank you for suggesting, sir. I've updated the figure. I hope this is better ..
              $endgroup$
              – Song
              Jan 26 at 22:38




              $begingroup$
              @HenningMakholm Thank you for suggesting, sir. I've updated the figure. I hope this is better ..
              $endgroup$
              – Song
              Jan 26 at 22:38












              $begingroup$
              Much nicer this way.
              $endgroup$
              – Henning Makholm
              Jan 26 at 23:49




              $begingroup$
              Much nicer this way.
              $endgroup$
              – Henning Makholm
              Jan 26 at 23:49











              3












              $begingroup$

              I like the following way.



              Let $vecAB=veca$, $vecAD=vecb$, $vecBF=pvecBD$ and $vecBC=kvecAD.$



              Thus, $$vecFE=-p(-veca+vecb)-veca+frac12vecb=(p-1)veca+left(frac12-pright)vecb$$ and
              $$vecFC=-p(-veca+vecb)+kvecb=pveca+(k-p)vecb.$$



              Now, we obtain the following system:
              $$|vecFE|=|vecFC|$$ and
              $$fracvecFEcdot vecFC=frac12$$
              with variables $p$ and $k$.



              We can solve this system and the rest is smooth.






              share|cite|improve this answer









              $endgroup$

















                3












                $begingroup$

                I like the following way.



                Let $vecAB=veca$, $vecAD=vecb$, $vecBF=pvecBD$ and $vecBC=kvecAD.$



                Thus, $$vecFE=-p(-veca+vecb)-veca+frac12vecb=(p-1)veca+left(frac12-pright)vecb$$ and
                $$vecFC=-p(-veca+vecb)+kvecb=pveca+(k-p)vecb.$$



                Now, we obtain the following system:
                $$|vecFE|=|vecFC|$$ and
                $$fracvecFEcdot vecFC=frac12$$
                with variables $p$ and $k$.



                We can solve this system and the rest is smooth.






                share|cite|improve this answer









                $endgroup$















                  3












                  3








                  3





                  $begingroup$

                  I like the following way.



                  Let $vecAB=veca$, $vecAD=vecb$, $vecBF=pvecBD$ and $vecBC=kvecAD.$



                  Thus, $$vecFE=-p(-veca+vecb)-veca+frac12vecb=(p-1)veca+left(frac12-pright)vecb$$ and
                  $$vecFC=-p(-veca+vecb)+kvecb=pveca+(k-p)vecb.$$



                  Now, we obtain the following system:
                  $$|vecFE|=|vecFC|$$ and
                  $$fracvecFEcdot vecFC=frac12$$
                  with variables $p$ and $k$.



                  We can solve this system and the rest is smooth.






                  share|cite|improve this answer









                  $endgroup$



                  I like the following way.



                  Let $vecAB=veca$, $vecAD=vecb$, $vecBF=pvecBD$ and $vecBC=kvecAD.$



                  Thus, $$vecFE=-p(-veca+vecb)-veca+frac12vecb=(p-1)veca+left(frac12-pright)vecb$$ and
                  $$vecFC=-p(-veca+vecb)+kvecb=pveca+(k-p)vecb.$$



                  Now, we obtain the following system:
                  $$|vecFE|=|vecFC|$$ and
                  $$fracvecFEcdot vecFC=frac12$$
                  with variables $p$ and $k$.



                  We can solve this system and the rest is smooth.







                  share|cite|improve this answer












                  share|cite|improve this answer



                  share|cite|improve this answer










                  answered Jan 26 at 15:55









                  Michael RozenbergMichael Rozenberg

                  103k1891195




                  103k1891195





















                      2












                      $begingroup$

                      Let $alpha=angle DEC$. We can apply the sine law to triangle $FED$:
                      $$
                      EDoversin(90°-alpha)=EFoversin30°=FDoversin(alpha+60°),
                      $$

                      that is:
                      $$
                      EF=1overcosalphaquadtextandquad FD=2overcosalphasin(alpha+60°).
                      $$

                      Applying then the sine law to triangle $BFC$ one gets:
                      $$
                      FB=2overcosalphasin(alpha-60°)=4sqrt3-FD=4sqrt3-2overcosalphasin(alpha+60°).
                      $$

                      From this it follows $tanalpha=2sqrt3$ and $EF^2=1/cos^2alpha=1+tan^2alpha=13$.






                      share|cite|improve this answer









                      $endgroup$

















                        2












                        $begingroup$

                        Let $alpha=angle DEC$. We can apply the sine law to triangle $FED$:
                        $$
                        EDoversin(90°-alpha)=EFoversin30°=FDoversin(alpha+60°),
                        $$

                        that is:
                        $$
                        EF=1overcosalphaquadtextandquad FD=2overcosalphasin(alpha+60°).
                        $$

                        Applying then the sine law to triangle $BFC$ one gets:
                        $$
                        FB=2overcosalphasin(alpha-60°)=4sqrt3-FD=4sqrt3-2overcosalphasin(alpha+60°).
                        $$

                        From this it follows $tanalpha=2sqrt3$ and $EF^2=1/cos^2alpha=1+tan^2alpha=13$.






                        share|cite|improve this answer









                        $endgroup$















                          2












                          2








                          2





                          $begingroup$

                          Let $alpha=angle DEC$. We can apply the sine law to triangle $FED$:
                          $$
                          EDoversin(90°-alpha)=EFoversin30°=FDoversin(alpha+60°),
                          $$

                          that is:
                          $$
                          EF=1overcosalphaquadtextandquad FD=2overcosalphasin(alpha+60°).
                          $$

                          Applying then the sine law to triangle $BFC$ one gets:
                          $$
                          FB=2overcosalphasin(alpha-60°)=4sqrt3-FD=4sqrt3-2overcosalphasin(alpha+60°).
                          $$

                          From this it follows $tanalpha=2sqrt3$ and $EF^2=1/cos^2alpha=1+tan^2alpha=13$.






                          share|cite|improve this answer









                          $endgroup$



                          Let $alpha=angle DEC$. We can apply the sine law to triangle $FED$:
                          $$
                          EDoversin(90°-alpha)=EFoversin30°=FDoversin(alpha+60°),
                          $$

                          that is:
                          $$
                          EF=1overcosalphaquadtextandquad FD=2overcosalphasin(alpha+60°).
                          $$

                          Applying then the sine law to triangle $BFC$ one gets:
                          $$
                          FB=2overcosalphasin(alpha-60°)=4sqrt3-FD=4sqrt3-2overcosalphasin(alpha+60°).
                          $$

                          From this it follows $tanalpha=2sqrt3$ and $EF^2=1/cos^2alpha=1+tan^2alpha=13$.







                          share|cite|improve this answer












                          share|cite|improve this answer



                          share|cite|improve this answer










                          answered Jan 26 at 16:30









                          AretinoAretino

                          23.6k21443




                          23.6k21443



























                              draft saved

                              draft discarded
















































                              Thanks for contributing an answer to Mathematics Stack Exchange!


                              • Please be sure to answer the question. Provide details and share your research!

                              But avoid


                              • Asking for help, clarification, or responding to other answers.

                              • Making statements based on opinion; back them up with references or personal experience.

                              Use MathJax to format equations. MathJax reference.


                              To learn more, see our tips on writing great answers.




                              draft saved


                              draft discarded














                              StackExchange.ready(
                              function ()
                              StackExchange.openid.initPostLogin('.new-post-login', 'https%3a%2f%2fmath.stackexchange.com%2fquestions%2f3088383%2fis-there-a-possible-geometric-method-to-find-length-of-this-equilateral-triangle%23new-answer', 'question_page');

                              );

                              Post as a guest















                              Required, but never shown





















































                              Required, but never shown














                              Required, but never shown












                              Required, but never shown







                              Required, but never shown

































                              Required, but never shown














                              Required, but never shown












                              Required, but never shown







                              Required, but never shown






                              Popular posts from this blog

                              How to check contact read email or not when send email to Individual?

                              Bahrain

                              Postfix configuration issue with fips on centos 7; mailgun relay